Download as pptx, pdf, or txt
Download as pptx, pdf, or txt
You are on page 1of 17

Investment

Management
Portfolio Performance Evaluation
Risk Adjusted Return
 Asset managers are targeted on return and risk
 So simply comparing asset returns does not lead
to meaningful evaluation
 Returns must be adjusted for relevant risk
 Risk adjusted return must be compared to
benchmark (i.e., index) risk adjusted return to
evaluate performance
 Risk adjusted returns must be ranked to
compare assets
Methods of Risk Adjustment
 Market Risk (SML) Based
 Used when asset is being evaluated as addition
to a well-diversified portfolio
 Includes Jensen’s Alpha and Treynor Ratio
 Total Risk (CML) Based
 Used when asset is being evaluated as stand-
alone investment
 Includes Sharpe Ratio and M2 (Modigliani and
Modigliani) Measure
Jensen’s Alpha

Jensen' s Alpha   P  rP  [rf   P (rM  rf )]

 Indicates by how much portfolio outperformed


SML equilibrium return
Treynor Ratio
rP  rf
Treynor Ratio 
P

 Measures return-to-risk, i.e., slope of line


from rf to (βP,rP) on SML chart
 Should be compared with benchmark return-
to-risk which is equal to SML slope
 Indicates if asset return-to-risk outperformed
benchmark return-to-risk
Market Risk Based Measures
 Both measures indicate asset position
relative to SML and will always indicate same
relative performance versus benchmark
 But they may rank assets differently because
 Jensen’s Alpha measures distance from SML
 Treynor Ratio measures slope
Sharpe Ratio
rP  rf
Sharpe Ratio 
P

 Measures return to total risk


 i.e., slope of line from rf to (σP, rP), i.e., CAL to
the portfolio
 Should be compared with benchmark Sharpe
ratio, i.e., slope of line from rf to (σM, rM),
which is equal to CML slope
M2
 Finds what portfolio
return would be if it had
same risk as benchmark
 Line joining rf and rP in
CML chart is
 rP  rf 
ri  rf    i
 P 
 If portfolio risk σP=σM its
return would have been
 rP  rf 
*
r  rf  
P
 M
 P 

 M2 = rP* - rM
Total Risk Based Measures
 Both always give same relative performance
versus benchmark as well as ranking
 Because both are slopes on the same chart
 Ranking will also be same because Sharpe
Ratio is integral to M2
*
rP  rf
ri  rf   M  rf  Sharpe Ratio P   M
P

 Since rf and σM are same for all assets, the M2


rankings are also same
Problem
Evaluate portfolios A and B from the point of view
of an investor who has (a) a well-diversified
portfolio (b) no other investments.

  Return Beta SD
Riskless 2.23%    
Index 4.88% 1.00 18.0%
Portfolio A 3.93% 1.13 12.0%
Portfolio B 4.86% 1.50 31.0%
Solution (a)
Jensen' s Alpha :
Index : 4.88%  [2.23%  1.00(4.88%  2.23%)]  0.00% (Rank  1)
Portfolio A : 3.93%  [2.23%  1.13( 4.88%  2.23%)]  1.29% (Rank  2)
Portfolio B : 4.86%  [2.23%  1.50(4.88%  2.23%)]  1.35% (Rank  3)
Treynor Ratio :
4.88%  2.23%
Index :  0.027 (Rank  1)
1.00
3.93%  2.23%
Portfolio A :  0.015 (Rank  3)
1.13
4.86%  2.23%
Portfolio B :  0.018 (Rank  2)
1.50
Conclusion : Both measures indicate that both portfolios underperformed the
index on a market risk adjusted basis but they are ranked differently
Solution (b)
Sharpe Ratio :
4.88%  2.23%
Index :  0.147 (Rank  1)
18.0%
3.93%  2.23%
Portfoio A :  0.142 (Rank  2)
12.0%
4.86%  2.23%
Portfolio B :  0.085 (Rank  3)
31.0%
M2 :
 4.88%  2.23% 
Index :  2.23%   18%   4.88%  0.00% (Rank  1)
 18.0% 
 3.93%  2.23% 
Portfoio A : 2.23%  18%  4.88%  0.10% (Rank  2)
 12.0% 
 4.86%  2.23% 
Portfolio B : 2.23%   18%  4.88%  1.12% (Rank  3)
 31.0% 
Conclusion : Both measures indicate that both portfolios underperformed the
index on a total risk adjusted basis and they are also ranked the same.
Performance Attribution
 Pinpoints which decisions resulted in
outperformance
 Return difference between a managed
portfolio and benchmark is the sum of
contributions made at various levels
 Attribution decomposes returns as due to
 Asset class allocation choice decision
 Security selection within each asset class
Problem
Following is performance of a fund manager versus his
benchmark in a recent month.
 Asset Manager Manager Benchmark Benchmark
Class Weight Return Weight Return
Equity 0.70 2.0% 0.60 2.5%
Bonds 0.20 1.0% 0.30 1.2%
Cash 0.10 0.5% 0.10 0.5%

a) How much was over/under performance?


b) What was security selection contribution to performance?
c) What was asset allocation contribution to performance?
Solution (a) and (b)
(a) Benchmark : (0.6  2.5%)  (0.3 1.2%)  (0.1 0.5%)  1.91%
Manager : (0.7  2.0%)  (0.2 1.0%)  (0.1 0.5%)  1.65%
Underperformance  0.26%

(b) Security Selection Contribution

Asset Manager Return  Manager Performance


Class Benchmark Return Weight Contribution
(1) ( 2) (1)  (2)
Equity  0.5% 0.7  0.35%
Bonds  0.2% 0.2  0.04%
Cash 0.0% 0.1 0.00%
Security Selection Contribution  0.39%
Solution (c)
The End

You might also like